Splitting a vector into a rowspace component and a nullspace component

Click For Summary
To find a basis for the orthogonal complement of the row space of matrix A, the reduced row echelon form (RREF) was calculated, yielding a basis for the nullspace as {(-2,-2,1)}. The discussion then shifted to splitting the vector x = (3,3,3) into components from the row space and nullspace. The approach involves expressing x as a combination of a vector from the nullspace and vectors from the row space, leading to a system of equations. The final step is to solve the resulting equations to determine the coefficients that represent the components of x. This method effectively breaks down the vector into its respective components.
starcoast
Messages
8
Reaction score
0

Homework Statement


Find a basis for the orthogonal complement of the row space of A:
A =
[1 0 2
1 1 4]

Split x = (3,3,3) into a row space component xr and a nullspace component xn.


The Attempt at a Solution


For the first part of the problem I took A to RREF
R =
[1 0 2
0 1 2]
and then solved to find a basis for the nullspace, x * (-2,-2,1), which should be the basis for the orthogonal complement of the row space.

I THINK that's right but maybe it's not because I'm stuck on the second part, splitting x = (3,3,3). Do I try to find a vector in Row(A) and a vector in Nul(A) that, when added, produce the vector (3,3,3)? How do I solve for that?

Help is very much appreciated!

Also, what's the best way to represent a matrix on these message boards?
 
Physics news on Phys.org
nevermind, I figured it out
 
an observation:

x(-2,-2,1) isn't the basis for Null(A), it's the entire space. "a" basis, is any particular vector, using a non-zero value for x. x = 1 works well, so one basis is:

{(-2,-2,1)}.

to find xn and xr, we're looking for a,b, and c with:

(3,3,3) = a(-2,-2,1) + (b(1,0,2) + c(1,1,4))

the first term is in Null(A) and the second sum of two terms in in Row(A).

this isn't that hard, we have:

(3,3,3) = (b-2a+c,c-2a,a+2b+4c), or, if you prefer:

\begin{bmatrix}-2&1&1\\-2&0&1\\1&2&4\end{bmatrix} \begin{bmatrix}a\\b\\c\end{bmatrix} = \begin{bmatrix}3\\3\\3\end{bmatrix}

solve this for a,b and c.

****
 
Last edited by a moderator:
Question: A clock's minute hand has length 4 and its hour hand has length 3. What is the distance between the tips at the moment when it is increasing most rapidly?(Putnam Exam Question) Answer: Making assumption that both the hands moves at constant angular velocities, the answer is ## \sqrt{7} .## But don't you think this assumption is somewhat doubtful and wrong?

Similar threads

  • · Replies 7 ·
Replies
7
Views
1K
  • · Replies 1 ·
Replies
1
Views
1K
Replies
7
Views
2K
  • · Replies 9 ·
Replies
9
Views
2K
  • · Replies 5 ·
Replies
5
Views
3K
  • · Replies 2 ·
Replies
2
Views
2K
Replies
1
Views
4K
  • · Replies 29 ·
Replies
29
Views
3K
  • · Replies 16 ·
Replies
16
Views
3K
  • · Replies 10 ·
Replies
10
Views
26K